How to integrate this one P(x1<x2<x3<1)

You can't use 0 as the lower limit of an integral over a variable that the upper limit of an earlier integral depends on.In summary, the joint pdf of three independent variables X1, X2, and X3 is given by f(x1, x2, x3) = e-(x1+x2+x3) for 0<x1,2,3<infinity and zero elsewhere. To compute the probability that X1<X2<X3 given X3<1, we use the formula P(X1< X2< X3|X3<1 )=P(X1< X2< X3<1 )/ P(X3<1). This can be calculated by taking a triple integral
  • #1
Jon08
3
0

Homework Statement


Let f(x1, x2, x3) = e-(x1+x2+x3), 0<x1,2,3<infinity, zero elsewhere be a joint pdf of X1, X2, X3. The variables are all independent to each other

Compute P(X1< X2< X3|X3<1 )

Homework Equations



P(X1< X2< X3|X3<1 )

The Attempt at a Solution


P(X1< X2< X3|X3<1 )=P(X1< X2< X3<1 )/ P(X3<1)
=triple integral of f(x1, x2, x3) dx1dx2dx3 as x1 goes from 0 to 1,x2 goes from x1 to x3, and x3 goes from x2 to 1.
 
Physics news on Phys.org
  • #2
Welcome to PF;
How come x1 does not go from 0 to x2?

Do you not know how to do ##\int e^x\;dx##?
 
  • #3
wait let me see my solution again.thanks
 
  • #4
Jon08 said:

Homework Statement


Let f(x1, x2, x3) = e-(x1+x2+x3), 0<x1,2,3<infinity, zero elsewhere be a joint pdf of X1, X2, X3. The variables are all independent to each other

Compute P(X1< X2< X3|X3<1 )

Homework Equations



P(X1< X2< X3|X3<1 )

The Attempt at a Solution


P(X1< X2< X3|X3<1 )=P(X1< X2< X3<1 )/ P(X3<1)
=triple integral of f(x1, x2, x3) dx1dx2dx3 as x1 goes from 0 to 1,x2 goes from x1 to x3, and x3 goes from x2 to 1.

Hint 1: what is ##P( \max(X_1,X_2,X_3) < 1)##?
Hint 2: what is the relation of the quantity in Hint 1 to the desired quantity ##P(X_1 < X_2 < X_3 < 1)##?
 
  • #5
Simon Bridge said:
Welcome to PF;
How come x1 does not go from 0 to x2?

Do you not know how to do ##\int e^x\;dx##?
x1 goes from 0 to x2,x2 goes from x1 to x3, and x3 goes from x2 to 1. Is my limits for the integration correct? Thanks
 
  • #6
Simon Bridge said:
Welcome to PF;
How come x1 does not go from 0 to x2?
Because, as stated in the same line, x2 goes from x1 to x3.
Jon08, when you list the ranges in a multiple integral, you need them in a sequence such that no integral range refers to the variable of an earlier range. I.e. if you integrate in the order x3, x2, x1 then the range for x1 cannot refer to x2 or x3, and that for x2 cannot refer to x3.
Other than that, you can generally integrate in any order you like, but the conditional x3 < 1 makes some orders better than others here.
 
  • #7
The x2<x3 is covered by putting x3 as the upper limit of integration on x2. You do not need to restate x2 as the lower limit of integration for x3. Same argument for x1. You are okay to use 0 as the lower limit on all three integrals.
 
  • #8
RUber said:
You are okay to use 0 as the lower limit on all three integrals.
... so long as the order is x1, x2, x3, right?
 
  • #9
Correct, I assumed the order of smallest to largest.
 

1. How do I approach integrating P(x1

As a scientist, you would approach this integration using mathematical methods and principles such as probability theory and calculus. It is important to carefully define the variables and understand the underlying distribution of the variables in order to accurately integrate the function.

2. Can I use any integration method to solve P(x1

Yes, there are various integration methods that can be used to solve this function, such as the trapezoidal rule, Simpson's rule, and Monte Carlo integration. The choice of method will depend on the specific parameters and distribution of the variables.

3. Is it necessary to have a specific range for the variables in order to integrate P(x1

Yes, the variables must have a defined range in order to integrate this function. It is important to carefully define the range of each variable and ensure that they are within the bounds of the function in order to get an accurate result.

4. How can I interpret the result of integrating P(x1

The result of integrating this function represents the probability of the variable x1 being less than x2, which is less than x3, which is less than 1. This can be interpreted as the likelihood of a specific event occurring based on the distribution of the variables.

5. Are there any limitations to integrating P(x1

As with any mathematical function, there may be limitations to integrating P(x1

Similar threads

  • MATLAB, Maple, Mathematica, LaTeX
Replies
5
Views
999
  • Calculus and Beyond Homework Help
Replies
4
Views
1K
  • Engineering and Comp Sci Homework Help
Replies
6
Views
3K
  • Calculus and Beyond Homework Help
Replies
4
Views
1K
Replies
5
Views
362
  • Calculus and Beyond Homework Help
Replies
8
Views
1K
  • Calculus and Beyond Homework Help
Replies
3
Views
3K
  • Calculus and Beyond Homework Help
Replies
2
Views
2K
  • Programming and Computer Science
Replies
1
Views
1K
  • Calculus and Beyond Homework Help
Replies
14
Views
1K

Back
Top